Wie viel Wasser muss durch den Kanal fließen, um eine konstante Wassertiefe aufrechtzuerhalten?

Wie viel Wasser muss durch das Rohr fließen, um ein konstantes Wasser tief im Kanal aufrechtzuerhalten?Geben Sie hier die Bildbeschreibung ein

Wie auf dem Bild gezeigt, hat der Kanal eine rechteckige Form. Ich weiß nicht, ob die Kanallänge einen Einfluss hat.

BEARBEITEN: Um die Dinge zu vereinfachen, nehmen wir an, dass es keine Turbulenzen und keine Viskosität gibt und dass aus dem Rohr fallendes Wasser das Wasser im Kanal nicht stört.


Ich habe versucht, das Problem selbst zu lösen (ich bin ein Physikanfänger, also könnte es völlig falsch sein, bitte stimmen Sie die Frage nicht ab, wenn Sie denken, dass dies nicht richtig ist):

Fläche des Kanalabschnitts: A = w H

Wenn ich die Wassergeschwindigkeit berechne v im Kanal, mit diesem und Oberfläche A , ich kann berechnen, wie viel Wasser Q wird fließen :

Q = A v

und löse das problem...

Also bleibt nur noch zu rechnen v .

Nehmen wir an, der Kanal hat keine Neigung Z = 0 , ich denke, die Wassergeschwindigkeit für eine bestimmte Wasserhöhe kann wie folgt berechnet werden (ich bin mir nicht sicher):

v = 2 G H

δ Q = A v = w δ H 2 G H

Integration von h von 0 bis H und ergibt:

Q = w 2 G 0 H H 1 / 2 D H

so Entladung für eine gegebene Höhe und Breite:

Q = 2 3 w 2 G H 3 / 2

Könnte mir jemand sagen, ob das obige richtig ist (vorausgesetzt, es gibt keine Neigung), und versuchen, meine ursprüngliche Frage zu beantworten?

Ihr Ausdruck für v ist nicht korrekt, da das Gesetz von Bernouilli nur entlang Stromlinien gültig ist.
Hallo Tigrou und willkommen bei Physics Stack Exchange! Ich füge das Hausaufgaben- Tag hinzu, weil Ihre Frage wie eine Hausaufgaben-ähnliche Frage aussieht (nicht unbedingt von einer tatsächlichen Hausaufgabe). Könnten Sie klären, ob dies der Fall ist oder warum Sie diese Frage stellen? Beachten Sie, dass es nichts Falsches daran gibt, dass es sich um eine Hausaufgabenfrage handelt! Wir sind nur daran interessiert zu wissen, welche Art von Antwort für Sie am nützlichsten ist.
Hi, erstmal ist das keine Hausaufgabe. Ich habe mich kürzlich für Fluiddynamik und Ingenieurwesen interessiert und einige Informationen darüber gelesen. Das ist eine Frage, die ich seit langem suche, ohne eine Antwort zu finden. Ich würde gerne wissen, ob das Prinzip von Bernoulli hier angewendet werden kann und ob es eine einfache Antwort auf dieses Problem gibt. Vielleicht fehlen einige Bedingungen. Personen, die versuchen, die Frage zu beantworten, können einige Bedingungen ändern/hinzufügen (oder einige Details vernachlässigen), wenn dies die Lösung des Problems erleichtert.
@tigrou Warum hast du diese Frage zum Community-Wiki gemacht?

Antworten (2)

Aufgrund der möglichen Anwendung der Frage gehe ich davon aus, dass die Frage Konstruktionsparameter für eine Wasserrutsche liefern soll. Da die Wassertiefe durch die Beschleunigung verringert wird, ist die eigentliche Frage, wie dem Kanal Widerstand hinzugefügt werden kann. Widerstand kann durch die Unterbrechung durch die Verwendung von Drehungen und Wendungen bereitgestellt werden. Ich würde empirische Daten eines tatsächlichen Aufbaus für ein Modell generieren. Ein Wasserdurchflussmesser könnte zeigen, wie verschiedene Graddrehungen die Geschwindigkeitsreduzierung der Flüssigkeit beeinflussen. Die Verwendung einer Flüssigkeit mit niedrigerer Viskosität könnte helfen, die Größe des Rohrs für Modellierungszwecke zu verringern. Da die Flüssigkeit nicht unbedingt im Durchmesser eines Rohrs enthalten ist, sind gängige Gleichungen nicht anwendbar, aber jemand in diesem Forum kann etwas Nützliches im Modellierungsbereich liefern.

Du hast Recht. Ich habe diese Frage ursprünglich gestellt, weil ich mich gefragt habe, wie Bauherren von River-Rafting-Fahrten bestimmen, wie viel Wasser durch die Fahrt fließen muss, um eine bestimmte Tiefe aufrechtzuerhalten. Ich hätte wahrscheinlich erwähnen sollen, dass dies in OP ist. Wie Sie sagten, hat der Weg viele Kurven und Kurven, die es schwierig machen könnten, ihn zu berechnen. Vielleicht bauen sie zuerst ein kleineres Modell, oder basiert das hauptsächlich auf Erfahrung?

Ich denke, es gibt einen grundlegenden Unterschied zwischen dem geneigten Hang und einer ebenen Fläche.

Wenn Sie den geneigten Fall betrachten, haben Sie die Schwerkraft als treibende Kraft, um die Strömung nach unten zu beschleunigen. Wenn die Flüssigkeit beschleunigt, wissen Sie aus der Massenerhaltung, dass der Wert von H am Hang nimmt ab. Aber irgendwann gab es Reibung an der Wand, die eine weitere Beschleunigung verhinderte. Konzentrieren wir uns auf diesen weit stromabwärts gelegenen Punkt, wo

D H ( X ) D X = 0

Daher wissen wir das auch u ( X , j ) reduziert zu u ( j ) , nur eine Geschwindigkeitskomponente in Richtung parallel zum Boden, die sich nur in Richtung senkrecht zum Boden ändert.

Wir wissen aus Erfahrung, dass diese Art von laminarer Strömung ein Poisseuille-Profil ergibt, also gehen wir davon aus u ( j ) = A j 2 + B j + C und Randbedingungen.

  • Bei j = 0 , u ( 0 ) = 0
  • Bei j = H , D u D j = 0 , weil es an der freien Oberfläche keinen Schlupf gibt.
  • Wir wissen jetzt, dass die Masse also erhalten bleibt 0 H u ( j ) D j = Q mit Q die Durchflussmenge. Beachten Sie, dass Q Maße hat M 2 / S da dieser Fall zweidimensional ist.

Dies geben C = 0 , B = 2 H A Und A = 2 3 Q H 3 jeweils und damit

u ( j ) = 3 Q H 3 ( j H 1 2 j 2 )

Jetzt werden wir die Reibung entlang des Bodens mit der Schwerkraft auf die Masse ausgleichen. Dies gilt, da kein Druckgefälle wirkt (der Druck oben ist gleichmäßiger Umgebungsdruck).

Die Reibungskraft entlang des Bodens über eine Länge Δ X gleich μ D u D j ( 0 ) Δ X und die Schwerkraft in dieser Richtung gleich ist ρ H G Sünde a , Wo μ ist die Viskosität, ρ die Dichte u a der Neigungswinkel zur Horizontalen.

F R ich C T ich Ö N = G R A v ich T j μ 3 Q H 2 Δ X = ρ H Δ X G Sünde a H = 3 μ Q ρ G Sünde a 3

Aus diesem Ausdruck sehen Sie, dass mit zunehmender Durchflussrate die Höhe des Films zunimmt. Auch wenn Sie den Neigungswinkel verringern, nimmt die Filmdicke zu.

Schließlich erhöht eine Erhöhung der Viskosität die Filmhöhe. Das macht Sinn, wenn man bedenkt, dass bei mehr Reibung die gleiche Fläche mehr Masse aushalten kann.

Dies wurde als "keine Antwort" gekennzeichnet, und in seiner aktuellen Form schwankt es meiner Meinung nach am Rande: Es ist eher wie ein langer Kommentar. Abgesehen davon kann ich sehen, worauf Sie damit hinauswollen, und denke, dass die Hinzufügung einer kleinen quantitativen Diskussion dies erheblich verbessern könnte.
@dmckee Ich hatte das Hausaufgaben-Tag und diese Diskussion im Sinn meta.physics.stackexchange.com/questions/714/… Ich habe viele Schritte bereitgestellt, die Ihnen zumindest eine Antwort geben werden. Vielleicht kann ich meine Antwort später erweitern, aber nicht ohne die genaue Ableitung selbst vorzunehmen. Eigentlich habe ich meine Antwort ohne Stift oder Papier in der Nachbarschaft geschrieben, was es schwierig macht, mehr hinzuzufügen als ich.
@tigrou Beantwortet es deine Frage oder ist etwas unklar?
Es gibt zwei Dinge, die ich nicht sicher verstehe: 1) von v = ax ^ 2 + bx + c, wie haben Sie herausgefunden, dass c = 0, b = -2ha, a = -2/3 ...? (Sie integrieren v(x), aber dann?) 2) Sie sagten Schwerkraft = phg * sin(alpha) woher kommt diese Formel? (Ich verstehe, warum die Sünde hier ist und was sie tut, aber nicht den Rest).
@tigrou 1) Aus den drei Bedingungen, die ich oben gebe, v (0) = c = 0. v_x(h)=2ah+b=0, und dann das Integral. 2) Das ist die Schwerkraft. Es ist mg und M = ρ v = ρ H Δ X , da das System 2D ist.